Đến nội dung

Hình ảnh

Bất đẳng thức chuẩn bị cho kì thi THPTQG 2015-2016


  • Please log in to reply
Chủ đề này có 389 trả lời

#321
caybutbixanh

caybutbixanh

    Trung úy

  • Thành viên
  • 888 Bài viết

Bài tiếp :( Anh 25 minutes thông cảm , giờ em cũng chẳng biết đánh số bài là bao nhiêu nữa ..)

1,Cho $x,y,z$ thỏa $x^2+y^2+z^2=1.$ Tìm GTNN của $P=x^3+y^3+z^3-3xyz$

2,Cho $x,y,z$ không âm thỏa $x^2+y^2+z^2 =3.$ Tìm GTLN $A=xy+yz+xz+\frac{5}{x+y+z}$....


KẺ MẠNH CHƯA CHẮC ĐÃ THẮNG



MÀ KẺ THẮNG MỚI CHÍNH LÀ KẺ MẠNH!.



(FRANZ BECKEN BAUER)




ÔN THI MÔN HÓA HỌC TẠI ĐÂY.


#322
Bubble

Bubble

    Binh nhì

  • Thành viên
  • 13 Bài viết

Cho a, b, c là các số thực dương, a+b+c=4. Chứng minh rằng:

      $a^{\frac{3}{4}} +b^{\frac{3}{4}} +c^{\frac{3}{4}}> 2\sqrt{2}$



#323
Bubble

Bubble

    Binh nhì

  • Thành viên
  • 13 Bài viết

Cho a,b,c >0. Tìm giá trị lớn nhất của M= $\frac{a+b+c}{(4a^{2}+2b^{2}+1)(4c^{2}+3)}$ 



#324
Dinh Xuan Hung

Dinh Xuan Hung

    Thành viên nổi bật 2015

  • Thành viên nổi bật 2016
  • 1396 Bài viết

Cho a,b,c >0. Tìm giá trị lớn nhất của M= $\frac{a+b+c}{(4a^{2}+2b^{2}+1)(4c^{2}+3)}$ 

Ta có:$(4a^2+2b^2+1)(4c^2+3)=\left ( 4a^2+2b^2+\frac{1}{3}+\frac{2}{3} \right )$

$\left ( \frac{1}{3}+\frac{2}{3}+4c^2+2 \right )\geq \left ( 2a.\frac{1}{\sqrt{3}}+\sqrt{2}b.\frac{\sqrt{2}}{\sqrt{3}}+\frac{1}{\sqrt{3}}.2c+\frac{\sqrt{2}}{\sqrt{3}}.\sqrt{2} \right )^2$

$=\frac{4}{3}(a+b+c+1)^2\geq \frac{4}{3}.4(a+b+c).1$

$=\frac{16}{3}(a+b+c)$

$\Rightarrow M=\frac{a+b+c}{(4a^{2}+2b^{2}+1)(4c^{2}+3)}\leq \frac{3}{16}$

Dâu bằng xảy ra khi $\left\{\begin{matrix} a=\dfrac{1}{6} & & & \\ b=\dfrac{1}{3} & & & \\ c=\dfrac{1}{2} & & & \end{matrix}\right.$



#325
Bubble

Bubble

    Binh nhì

  • Thành viên
  • 13 Bài viết

Ta có:$(4a^2+2b^2+1)(4c^2+3)=\left ( 4a^2+2b^2+\frac{1}{3}+\frac{2}{3} \right )$

$\left ( \frac{1}{3}+\frac{2}{3}+4c^2+2 \right )\geq \left ( 2a.\frac{1}{\sqrt{3}}+\sqrt{2}b.\frac{\sqrt{2}}{\sqrt{3}}+\frac{1}{\sqrt{3}}.2c+\frac{\sqrt{2}}{\sqrt{3}}.\sqrt{2} \right )^2$

$=\frac{4}{3}(a+b+c+1)^2\geq \frac{4}{3}.4(a+b+c).1$

$=\frac{16}{3}(a+b+c)$

$\Rightarrow M=\frac{a+b+c}{(4a^{2}+2b^{2}+1)(4c^{2}+3)}\leq \frac{3}{16}$

Dâu bằng xảy ra khi $\left\{\begin{matrix} a=\dfrac{1}{6} & & & \\ b=\dfrac{1}{3} & & & \\ c=\dfrac{1}{2} & & & \end{matrix}\right.$

Bạn cho mình hỏi đoán dấu bằng như nào vậy??



#326
kimchitwinkle

kimchitwinkle

    Thiếu úy

  • Điều hành viên THCS
  • 526 Bài viết

(D- 2007) Cho $a\geq b> 0$ . CMR: $\left ( 2^{a} +\frac{1}{2^{a}}\right )^{b}\leq \left ( 2^{b} +\frac{1}{2^{b}}\right )^{a}$

(Dự bị A- 2005) CMR $\forall x,y> 0$ ta có : $(1+x)(1+\frac{y}{x})(1+\frac{y}{\sqrt{y}})^{2}\geq 256$

(B- 2005) CMR $\forall x\epsilon R$ ta có : $\left ( \frac{12}{5} \right )^{x}+\left ( \frac{15}{4} \right )^{x}+\left ( \frac{10}{3} \right )^{x}\geq 3^{x}+4^{x}+5^{x}$



#327
25 minutes

25 minutes

    Thành viên nổi bật 2015

  • Hiệp sỹ
  • 2795 Bài viết

Cho a, b, c là các số thực dương, a+b+c=4. Chứng minh rằng:

      $a^{\frac{3}{4}} +b^{\frac{3}{4}} +c^{\frac{3}{4}}> 2\sqrt{2}$

Đặt $x^4=a.y^4=b,z^4=c\Rightarrow x^4+y^4+z^4=4$

Ta cần chứng minh $x^3+y^3+z^3>2\sqrt{2}$

Do $x^4+y^4+z^4=4\Rightarrow x<\sqrt{2}\Rightarrow x^4< \sqrt{2}x^3$

$\Rightarrow \sum x^4<\sqrt{2}\sum x^3\Rightarrow x^3+y^3+z^3>\frac{4}{\sqrt{2}}=2\sqrt{2}$


Hãy theo đuổi đam mê, thành công sẽ theo đuổi bạn.



Thảo luận BĐT ôn thi Đại học tại đây


#328
25 minutes

25 minutes

    Thành viên nổi bật 2015

  • Hiệp sỹ
  • 2795 Bài viết

Bài tiếp :( Anh 25 minutes thông cảm , giờ em cũng chẳng biết đánh số bài là bao nhiêu nữa ..)

1,Cho $x,y,z$ thỏa $x^2+y^2+z^2=1.$ Tìm GTNN của $P=x^3+y^3+z^3-3xyz$

2,Cho $x,y,z$ không âm thỏa $x^2+y^2+z^2 =3.$ Tìm GTLN $A=xy+yz+xz+\frac{5}{x+y+z}$....

Bài 1: Ta có $P=\frac{1}{2}(x+y+z)[(x-y)^2+(y-z)^2+(z-x)^2]=\frac{1}{2}(x+y+z)[2-2(xy+yz+zx)]=(x+y+z)[1-(xy+yz+zx)]$

Đặt $t=x+y+z\Rightarrow xy+yz+zx=\frac{t^2-1}{2}\Rightarrow P=t.(1-\frac{t^2-1}{2})=f(t)$

Với điều kiện $t \in [1;\sqrt{3}]$

Bài 2: Đặt $t=x+y+z\Rightarrow A=\frac{t^2-3}{2}+\frac{5}{t}=f(t)$

Với điều kiện $t \in [\sqrt{3};3]$


Hãy theo đuổi đam mê, thành công sẽ theo đuổi bạn.



Thảo luận BĐT ôn thi Đại học tại đây


#329
quan1234

quan1234

    Thượng sĩ

  • Thành viên
  • 257 Bài viết

(D- 2007) Cho $a\geq b> 0$ . CMR: $\left ( 2^{a} +\frac{1}{2^{a}}\right )^{b}\leq \left ( 2^{b} +\frac{1}{2^{b}}\right )^{a}$

(Dự bị A- 2005) CMR $\forall x,y> 0$ ta có : $(1+x)(1+\frac{y}{x})(1+\frac{y}{\sqrt{y}})^{2}\geq 256$

(B- 2005) CMR $\forall x\epsilon R$ ta có : $\left ( \frac{12}{5} \right )^{x}+\left ( \frac{15}{4} \right )^{x}+\left ( \frac{10}{3} \right )^{x}\geq 3^{x}+4^{x}+5^{x}$

Câu B-2005 

$(\frac{12}{5})^x+(\frac{15}{4})^x+(\frac{15}{4})^x+(\frac{10}{3})^x+(\frac{10}{3})^x+(\frac{12}{5})^x\geq 2(3^x+4^x+5^x)\Rightarrow (\frac{12}{5})^x+(\frac{15}{4})^x+(\frac{10}{3})^x\geq 3^x+4^x+5^x$



#330
25 minutes

25 minutes

    Thành viên nổi bật 2015

  • Hiệp sỹ
  • 2795 Bài viết

(D- 2007) Cho $a\geq b> 0$ . CMR: $\left ( 2^{a} +\frac{1}{2^{a}}\right )^{b}\leq \left ( 2^{b} +\frac{1}{2^{b}}\right )^{a}$

(B- 2005) CMR $\forall x\epsilon R$ ta có : $\left ( \frac{12}{5} \right )^{x}+\left ( \frac{15}{4} \right )^{x}+\left ( \frac{10}{3} \right )^{x}\geq 3^{x}+4^{x}+5^{x}$

D-2007: Chia cả 2 vế cho $2^{ab}$ ta được bất đẳng thức tương đương 

  $(1+\frac{1}{4^a})^b\leqslant (1+\frac{1}{4^b})^a$

BĐT trên luôn đúng do $(1+\frac{1}{4^a})^b\leqslant (1+\frac{1}{4^a})^a\leqslant  (1+\frac{1}{4^b})^a$

B-2005: Sử dụng AM-GM ta có ngay 

    $(\frac{12}{5})^x+(\frac{15}{4})^x\geqslant 2.3^x$

Tương tự 2 bất đẳng thức còn lại

P/S: Những bài toán đã thi thì không nên post lại, tránh gây loãng.


Hãy theo đuổi đam mê, thành công sẽ theo đuổi bạn.



Thảo luận BĐT ôn thi Đại học tại đây


#331
Pino

Pino

    Hạ sĩ

  • Thành viên
  • 84 Bài viết

Bài ....: Chứng minh rằng: $x+\sqrt{2(x^2-x+1)} \geq 1+\sqrt{x}$   $\forall x \geq 0$


~~  :ukliam2:  :ukliam2:  :ukliam2:  :luoi  :luoi  $\boxed{\boxed{\bigstar \bigstar\text{PINO}\bigstar \bigstar}}$  :luoi  :luoi  :ukliam2:  :ukliam2:  :ukliam2: ~~


#332
buitudong1998

buitudong1998

    Trung úy

  • Thành viên
  • 873 Bài viết

Bài ....: Chứng minh rằng: $x+\sqrt{2(x^2-x+1)} \geq 1+\sqrt{x}$   $\forall x \geq 0$

Xét trường hợp $x\geqslant 1+\sqrt{x}$ thì BĐT hiển nhiên đúng

Trường hợp còn lại: $x\leqslant 1+\sqrt{x}$, đặt $\sqrt{x}=t\geqslant 0$, chuyển vế bình phương và rút gọn ta được:

$BDT\Leftrightarrow t^{4}+2t^{3}-t^{2}-2t+1\geqslant 0\Leftrightarrow (t^{2}+t-1)^{2}\geqslant 0$ (Luôn đúng), vậy trong cả hai trường hợp ta đều có ĐPCM


Đứng dậy và bước tiếp

#333
buitudong1998

buitudong1998

    Trung úy

  • Thành viên
  • 873 Bài viết

Mở rộng của bài trên: Cho $x,y,z$ dương và $x+y+z=3$. Tìm giá trị nhỏ nhất của biểu thức:

$P=\frac{x}{2+\sqrt{2x^{2}+4x-5}}+\frac{y}{2+\sqrt{2y^{2}+4y-5}}+\frac{z}{2+\sqrt{2z^{2}+4z-5}}$


Bài viết đã được chỉnh sửa nội dung bởi buitudong1998: 24-08-2015 - 23:05

Đứng dậy và bước tiếp

#334
tranquocviet97

tranquocviet97

    Lính mới

  • Thành viên
  • 7 Bài viết

k2pi.net.vn-240215.JPG



#335
Bubble

Bubble

    Binh nhì

  • Thành viên
  • 13 Bài viết

Cho x,y,z là các số thực dương. xyz=1. Tìm GTNN của

P = $\sum \frac{x^{2}(y+z)}{y\sqrt{y}+z\sqrt{z}}$



#336
tranquocviet97

tranquocviet97

    Lính mới

  • Thành viên
  • 7 Bài viết

 

Bài 66:
 
Cho $ x,y,z > 0$ thỏa mãn $4(x^{2}-x+1)\leq 16\sqrt{x^{2}yz}-3x(y+z)^{2}$.Tìm MIN:
$P=\frac{y+3x^{2}+3x}{x^{2}z}+\frac{16}{(y+1)^{3}}-10\sqrt{3}\sqrt{\frac{y}{x^{3}+2}}$
 
 

  

Bài này đề Lương Thế Vinh-Hà Nội lần 4,dồn về hàm số $f(\dfrac{y}{x})$ 

Bài viết đã được chỉnh sửa nội dung bởi tranquocviet97: 31-08-2015 - 13:36


#337
Dinh Xuan Hung

Dinh Xuan Hung

    Thành viên nổi bật 2015

  • Thành viên nổi bật 2016
  • 1396 Bài viết

Bài 8 Cho a,b,c là các số dương thỏa mãn a+2b-c>0 và $a^2+b^2+c^2=ab+bc+ac+2$.Tìm GTLN của biểu thức: 

P=$\frac{a+c+2}{a(b+c)+a+b+1}-\frac{a+b+1}{(a+c)(a+2b-c)}$

Ta có:$ab+bc+ac+2=a^2+b^2+c^2\geq a^2+2bc$

 

$\Leftrightarrow ab+ac+2\geq a^2+bc$

 

$\Leftrightarrow 2(ab+ac)+2\geq (a+b)(a+c)$

 

$\Leftrightarrow a(b+c)+1\geq \frac{(a+b)(a+c)}{2}$

 

$\Leftrightarrow a(b+c)+a+b+1\geq \frac{(a+b)(a+c+2)}{2}$

 

$\Rightarrow \frac{a+c+2}{a(b+c)+a+b+1}\leq \frac{2}{a+b}$

 

Áp dụng bất đẳng thức $AM-GM$ ta có:

$(a+c)(a+2b-c)\leq \frac{(a+c+a+2b-c)^2}{4}=(a+b)^2\Rightarrow \frac{a+b+1}{(a+c)(a+2b-c)}\geq \frac{a+b+1}{(a+b)^2}$

 

$\Rightarrow P\leq \frac{2}{a+b}-\frac{a+b+1}{(a+b)^2}=\frac{1}{a+b}-\frac{1}{(a+b)^2}=\frac{1}{4}-\left ( \frac{1}{a+b}-\frac{1}{2} \right )^2\leq \frac{1}{4}$

 

Vậy P max=$1/4$



#338
caybutbixanh

caybutbixanh

    Trung úy

  • Thành viên
  • 888 Bài viết

Cho x,y,z là các số thực dương. xyz=1. Tìm GTNN của

P = $\sum \frac{x^{2}(y+z)}{y\sqrt{y}+z\sqrt{z}}$

Lời giải :

 Giả thiết $xyz=1$ có nhiều cách để xử lí, ở đây ta sẽ đánh giá làm sao để tử và mẫu có sự liên hệ "gần gũi" với nhau, cụ thể :

$$x^2(y+z) =x^2.y+x^2.z \geq 2.\sqrt{x^4yz}=2.x\sqrt{x};$$

Hoàn toàn tương tự, ta sẽ có:

$$P \geq \frac{2.x\sqrt{x}}{y.\sqrt{y}+z.\sqrt{z}}+\frac{2.y\sqrt{y}}{x.\sqrt{x}+z.\sqrt{z}}+\frac{2.z\sqrt{z}}{y.\sqrt{y}+x.\sqrt{x}}$$

Ta đặt $a=x.\sqrt{x};b=y.\sqrt{y};c=z.\sqrt{z}$, (a,b,c dương ) khi đó :

$$P \geq \frac{2a}{b+c}+\frac{2b}{a+c}+\frac{2c}{a+b};$$

mặt khác, để đánh giá biểu thức vế phải, ta xét biểu thức tổng quát hơn như sau :

Xét biểu thức  $\frac{a}{mb+nc}+\frac{b}{mc+na}+\frac{c}{ma+nb};$ với $m,n$ dương, khi đó :

 $$\frac{a}{mb+nc}+\frac{b}{mc+na}+\frac{c}{ma+nb}=\frac{a^2}{mba+nca}+\frac{b^2}{mab+ncb}+\frac{c^2}{mac+nbc} \geq \frac{(a+b+c)^2}{(m+n)(ab+bc+ca)} \geq \frac{3(ab+ca+bc)}{(m+n)(ab+bc+ca)}=\frac{3}{m+n}$$

Áp dụng ta có : $P \geq 2$ ( Giả thiết $abc=1$ lúc này cũng chẳng cần)

Dấu bằng xảy ra khi $x=y=z=1$

-------------------------------------


KẺ MẠNH CHƯA CHẮC ĐÃ THẮNG



MÀ KẺ THẮNG MỚI CHÍNH LÀ KẺ MẠNH!.



(FRANZ BECKEN BAUER)




ÔN THI MÔN HÓA HỌC TẠI ĐÂY.


#339
Bubble

Bubble

    Binh nhì

  • Thành viên
  • 13 Bài viết

Cho a,b,c là các số không âm trong đó không có 2 số nào đồng thời bằng 0. Chứng minh rằng: $\sum \frac{3a^{3}+abc}{b^{3}+c^{3}}\geq 6$



#340
longatk08

longatk08

    Sĩ quan

  • Thành viên
  • 350 Bài viết

Cho a,b,c là các số không âm trong đó không có 2 số nào đồng thời bằng 0. Chứng minh rằng: $\sum \frac{3a^{3}+abc}{b^{3}+c^{3}}\geq 6$

Đăng nhầm topic rồi bạn,những bài thế này nên chuyển sang mục BĐT toán THPT.

 

Về ý tưởng cho bài này ta sẽ thực hiện phân li đẳng thức vì ngoài dấu bằng đạt tại $a=b=c$ thì còn có $a=b,c=0$ hay các hoán vị của chúng. Cụ thể ta sẽ tách riêng đại lượng $abc$ ra một bên vì dấu bằng thứ 2 sinh ra là do đại lượng này, khi đó ta có thể thoải mái đánh giá lượng còn lại mà không cần quan tâm đến điểm rơi đó nữa.

 

BĐT của chúng ta tương đương với:

 

$\sum \frac{3a^3}{b^3+c^3}+abc.\sum \frac{1}{b^3+c^3} \geq 6$

 

Áp dụng BĐT AM-GM thì ta có: $\sum (a^3+b^3)(b^3+c^3)\geq 3\sqrt[3]{\prod (a^3+b^3)^2}\geq 12a^2b^2c^2$

 

Do đó ta cần chứng minh:

 

$\sum \frac{a^3}{b^3+c^3}+\frac{4a^3b^3c^3}{\prod (a^3+b^3)}\geq 2$

 

Đây chính là BĐT Schur dạng phân thức.






1 người đang xem chủ đề

0 thành viên, 1 khách, 0 thành viên ẩn danh